CAT Exam  >  CAT Tests  >  CAT Mock Test Series and 500+ Practice Tests 2024  >  CAT Practice Test - 26 - CAT MCQ

CAT Practice Test - 26 - CAT MCQ


Test Description

30 Questions MCQ Test CAT Mock Test Series and 500+ Practice Tests 2024 - CAT Practice Test - 26

CAT Practice Test - 26 for CAT 2024 is part of CAT Mock Test Series and 500+ Practice Tests 2024 preparation. The CAT Practice Test - 26 questions and answers have been prepared according to the CAT exam syllabus.The CAT Practice Test - 26 MCQs are made for CAT 2024 Exam. Find important definitions, questions, notes, meanings, examples, exercises, MCQs and online tests for CAT Practice Test - 26 below.
Solutions of CAT Practice Test - 26 questions in English are available as part of our CAT Mock Test Series and 500+ Practice Tests 2024 for CAT & CAT Practice Test - 26 solutions in Hindi for CAT Mock Test Series and 500+ Practice Tests 2024 course. Download more important topics, notes, lectures and mock test series for CAT Exam by signing up for free. Attempt CAT Practice Test - 26 | 100 questions in 180 minutes | Mock test for CAT preparation | Free important questions MCQ to study CAT Mock Test Series and 500+ Practice Tests 2024 for CAT Exam | Download free PDF with solutions
CAT Practice Test - 26 - Question 1

Group Question

The passage given below is followed by a set of questions. Choose the most appropriate answer to each question.

We live in a time when about 896 million people survive on less than $1.90 a day. A time, when the world produces sufficient food to provide 2700 k/cal to every person every day. Yet, a vast section of humanity is unable to afford it. The reason being this is the paucity of funds. Traditionally, the private sector has always been associated with the relentless drive to maximise profits, often at the cost of rising inequality. Whereas, the public sector is shown to pay greater heed to the distributive aspect of economic benefits. The private - public difference rests on the relative weights attached to efficiency and equity. While the private sector limits its arena of concern to the bottom line - subtraction of costs from revenues, the overarching role of the public sector is linked to enhancing welfare and promoting equity.
Poverty eradication has perks for the private sector too. Bottom of the Pyramid investment by multinational corporations, such as those pursued by Danone in India and Unilever in Africa, helps to lift billions out of poverty. This, in turn, allows deprived communities to access products and services that have been appropriately priced for them. Whatever the private sector loses in terms of margins, it gains in terms of volumes. This way, the private sector is able to tap into new markets and geographies. Further, participating in the development process allows the private sector to gain in terms of risk mitigation and greater value creation, in addition to positive social positioning.
Currently, Corporate Social Responsibility (CSR) initiatives by the private sector show promising results. Be it is the Tata Foundation supporting weavers in Varanasi or the Bill and Melinda Gates Foundation providing grants to fight Ebola in West Africa, the private sector is increasingly becoming part of a shared agenda to end global poverty. Private-public partnerships can also play a crucial role in elevating livelihoods. Recently, an IFAD-supported project in Rwanda forged a partnership between tea cooperatives and private investors- ensuring 30-40% equity shares for the cooperatives.
Further, private sector jobs are more diversified and generate higher wages. If higher incomes and job security is a marker of poverty alleviation, then the private sector is steadily contributing to reducing inequalities. It generates employment and makes livelihoods higher. In addition, there is immense potential for commercial credit to benefit the lives of the rural poor. While complete credit penetration into the rural hinterland and urban slums is yet to be seen, the availability of commercial credit will no doubt protect the poor against exorbitant interest rates and debt traps.
Concludingly, the private sector can end poverty. It can do so because of the new ideas it can bring to the table - innovation, strategy and skills. It has the immense resources it currently wields, and the humongous amount of influence that comes with it. The sector can participate because it is in its own interests to do so - in terms of opening up new markets and tapping new possibilities. All it needs is a reworking of the concept of ‘profit’ - from being a commercial end goal accruing to shareholders, to a social product to be distributed among all stakeholders.

 

Q. Which of the following is true about the private sector?

Detailed Solution for CAT Practice Test - 26 - Question 1

The passage states that “Traditionally, the private sector has always been associated with the relentless drive to maximise profits,..”, thus option 1 is contextually correct.
Option 2 is true with respect to the public sector, hence can be eliminated.
Options 3 and 4 are contextually misleading as the passage mentions that “Be it is the Tata Foundation supporting weavers in Varanasi or the Bill and Melinda Gates Foundation providing grants to fight Ebola in West Africa,...” Hence, the correct answer is option 1.

CAT Practice Test - 26 - Question 2

We live in a time when about 896 million people survive on less than $1.90 a day. A time, when the world produces sufficient food to provide 2700 k/cal to every person every day. Yet, a vast section of humanity is unable to afford it. The reason being this is the paucity of funds. Traditionally, the private sector has always been associated with the relentless drive to maximise profits, often at the cost of rising inequality. Whereas, the public sector is shown to pay greater heed to the distributive aspect of economic benefits. The private - public difference rests on the relative weights attached to efficiency and equity. While the private sector limits its arena of concern to the bottom line - subtraction of costs from revenues, the overarching role of the public sector is linked to enhancing welfare and promoting equity.
Poverty eradication has perks for the private sector too. Bottom of the Pyramid investment by multinational corporations, such as those pursued by Danone in India and Unilever in Africa, helps to lift billions out of poverty. This, in turn, allows deprived communities to access products and services that have been appropriately priced for them. Whatever the private sector loses in terms of margins, it gains in terms of volumes. This way, the private sector is able to tap into new markets and geographies. Further, participating in the development process allows the private sector to gain in terms of risk mitigation and greater value creation, in addition to positive social positioning.
Currently, Corporate Social Responsibility (CSR) initiatives by the private sector show promising results. Be it is the Tata Foundation supporting weavers in Varanasi or the Bill and Melinda Gates Foundation providing grants to fight Ebola in West Africa, the private sector is increasingly becoming part of a shared agenda to end global poverty. Private-public partnerships can also play a crucial role in elevating livelihoods. Recently, an IFAD-supported project in Rwanda forged a partnership between tea cooperatives and private investors- ensuring 30-40% equity shares for the cooperatives.
Further, private sector jobs are more diversified and generate higher wages. If higher incomes and job security is a marker of poverty alleviation, then the private sector is steadily contributing to reducing inequalities. It generates employment and makes livelihoods higher. In addition, there is immense potential for commercial credit to benefit the lives of the rural poor. While complete credit penetration into the rural hinterland and urban slums is yet to be seen, the availability of commercial credit will no doubt protect the poor against exorbitant interest rates and debt traps.
Concludingly, the private sector can end poverty. It can do so because of the new ideas it can bring to the table - innovation, strategy and skills. It has the immense resources it currently wields, and the humongous amount of influence that comes with it. The sector can participate because it is in its own interests to do so - in terms of opening up new markets and tapping new possibilities. All it needs is a reworking of the concept of ‘profit’ - from being a commercial end goal accruing to shareholders, to a social product to be distributed among all stakeholders.

 

Q. Which of the following would weaken the argument on ‘the importance of private sector in ending poverty’?

A. Investments by multinational corporations such as Wal-Mart in Qatar.
B. CSR initiatives by the private sector discharged through social welfare projects and schemes.
C. Private-public partnerships between the tea cooperatives and private investors.

Detailed Solution for CAT Practice Test - 26 - Question 2

Statement A weakens the argument as Qatar is known to be one of the richest country in the world, hence investments by a multinational corporations in this country would not really tackle the issue of poverty.
Statement B would definitely have a positive impact on the livelihood of the people, hence it strengthens the argument. Statement C is mentioned in the fourth paragraph of the passage and such a collaboration is shown to elevate the livelihood of people, hence it strengthens the argument.
Thus, only option 1 is apt.
Hence, the correct answer is option 1.

1 Crore+ students have signed up on EduRev. Have you? Download the App
CAT Practice Test - 26 - Question 3

We live in a time when about 896 million people survive on less than $1.90 a day. A time, when the world produces sufficient food to provide 2700 k/cal to every person every day. Yet, a vast section of humanity is unable to afford it. The reason being this is the paucity of funds. Traditionally, the private sector has always been associated with the relentless drive to maximise profits, often at the cost of rising inequality. Whereas, the public sector is shown to pay greater heed to the distributive aspect of economic benefits. The private - public difference rests on the relative weights attached to efficiency and equity. While the private sector limits its arena of concern to the bottom line - subtraction of costs from revenues, the overarching role of the public sector is linked to enhancing welfare and promoting equity.
Poverty eradication has perks for the private sector too. Bottom of the Pyramid investment by multinational corporations, such as those pursued by Danone in India and Unilever in Africa, helps to lift billions out of poverty. This, in turn, allows deprived communities to access products and services that have been appropriately priced for them. Whatever the private sector loses in terms of margins, it gains in terms of volumes. This way, the private sector is able to tap into new markets and geographies. Further, participating in the development process allows the private sector to gain in terms of risk mitigation and greater value creation, in addition to positive social positioning.
Currently, Corporate Social Responsibility (CSR) initiatives by the private sector show promising results. Be it is the Tata Foundation supporting weavers in Varanasi or the Bill and Melinda Gates Foundation providing grants to fight Ebola in West Africa, the private sector is increasingly becoming part of a shared agenda to end global poverty. Private-public partnerships can also play a crucial role in elevating livelihoods. Recently, an IFAD-supported project in Rwanda forged a partnership between tea cooperatives and private investors- ensuring 30-40% equity shares for the cooperatives.
Further, private sector jobs are more diversified and generate higher wages. If higher incomes and job security is a marker of poverty alleviation, then the private sector is steadily contributing to reducing inequalities. It generates employment and makes livelihoods higher. In addition, there is immense potential for commercial credit to benefit the lives of the rural poor. While complete credit penetration into the rural hinterland and urban slums is yet to be seen, the availability of commercial credit will no doubt protect the poor against exorbitant interest rates and debt traps.
Concludingly, the private sector can end poverty. It can do so because of the new ideas it can bring to the table - innovation, strategy and skills. It has the immense resources it currently wields, and the humongous amount of influence that comes with it. The sector can participate because it is in its own interests to do so - in terms of opening up new markets and tapping new possibilities. All it needs is a reworking of the concept of ‘profit’ - from being a commercial end goal accruing to shareholders, to a social product to be distributed among all stakeholders.

 

Q. What is the tone of the passage?

Detailed Solution for CAT Practice Test - 26 - Question 3

A didactic tone implies that the author is trying to teach/ introduce a new concept to the readers.
A sceptical tone implies that the author has a dismissive attitude towards the contents of the passage.
An analytical tone is used when the author is examining a particular situation.
A contemplative tone is used by the author who has carefully studied the issue in hand and his writing reflects this seriousness.
Throughout the passage, the author talks about how the coalition of working styles of the public and private sector can eradicate poverty. Thus, the passage presents a contemplative tone.
Hence, the correct answer is option 4.

CAT Practice Test - 26 - Question 4

We live in a time when about 896 million people survive on less than $1.90 a day. A time, when the world produces sufficient food to provide 2700 k/cal to every person every day. Yet, a vast section of humanity is unable to afford it. The reason being this is the paucity of funds. Traditionally, the private sector has always been associated with the relentless drive to maximise profits, often at the cost of rising inequality. Whereas, the public sector is shown to pay greater heed to the distributive aspect of economic benefits. The private - public difference rests on the relative weights attached to efficiency and equity. While the private sector limits its arena of concern to the bottom line - subtraction of costs from revenues, the overarching role of the public sector is linked to enhancing welfare and promoting equity.
Poverty eradication has perks for the private sector too. Bottom of the Pyramid investment by multinational corporations, such as those pursued by Danone in India and Unilever in Africa, helps to lift billions out of poverty. This, in turn, allows deprived communities to access products and services that have been appropriately priced for them. Whatever the private sector loses in terms of margins, it gains in terms of volumes. This way, the private sector is able to tap into new markets and geographies. Further, participating in the development process allows the private sector to gain in terms of risk mitigation and greater value creation, in addition to positive social positioning.
Currently, Corporate Social Responsibility (CSR) initiatives by the private sector show promising results. Be it is the Tata Foundation supporting weavers in Varanasi or the Bill and Melinda Gates Foundation providing grants to fight Ebola in West Africa, the private sector is increasingly becoming part of a shared agenda to end global poverty. Private-public partnerships can also play a crucial role in elevating livelihoods. Recently, an IFAD-supported project in Rwanda forged a partnership between tea cooperatives and private investors- ensuring 30-40% equity shares for the cooperatives.
Further, private sector jobs are more diversified and generate higher wages. If higher incomes and job security is a marker of poverty alleviation, then the private sector is steadily contributing to reducing inequalities. It generates employment and makes livelihoods higher. In addition, there is immense potential for commercial credit to benefit the lives of the rural poor. While complete credit penetration into the rural hinterland and urban slums is yet to be seen, the availability of commercial credit will no doubt protect the poor against exorbitant interest rates and debt traps.
Concludingly, the private sector can end poverty. It can do so because of the new ideas it can bring to the table - innovation, strategy and skills. It has the immense resources it currently wields, and the humongous amount of influence that comes with it. The sector can participate because it is in its own interests to do so - in terms of opening up new markets and tapping new possibilities. All it needs is a reworking of the concept of ‘profit’ - from being a commercial end goal accruing to shareholders, to a social product to be distributed among all stakeholders.

 

Q. Which of the following is the author most likely to agree with?

Detailed Solution for CAT Practice Test - 26 - Question 4

Option 1 is not mentioned in the passage, thus options 1 and 4 can be eliminated.
Option 3 is corroborated as the passage mentions “Concludingly, the private sector can end poverty.” In view of option 3, option 2 can be ruled out.
Hence, the correct answer is option 3.

CAT Practice Test - 26 - Question 5

We live in a time when about 896 million people survive on less than $1.90 a day. A time, when the world produces sufficient food to provide 2700 k/cal to every person every day. Yet, a vast section of humanity is unable to afford it. The reason being this is the paucity of funds. Traditionally, the private sector has always been associated with the relentless drive to maximise profits, often at the cost of rising inequality. Whereas, the public sector is shown to pay greater heed to the distributive aspect of economic benefits. The private - public difference rests on the relative weights attached to efficiency and equity. While the private sector limits its arena of concern to the bottom line - subtraction of costs from revenues, the overarching role of the public sector is linked to enhancing welfare and promoting equity.
Poverty eradication has perks for the private sector too. Bottom of the Pyramid investment by multinational corporations, such as those pursued by Danone in India and Unilever in Africa, helps to lift billions out of poverty. This, in turn, allows deprived communities to access products and services that have been appropriately priced for them. Whatever the private sector loses in terms of margins, it gains in terms of volumes. This way, the private sector is able to tap into new markets and geographies. Further, participating in the development process allows the private sector to gain in terms of risk mitigation and greater value creation, in addition to positive social positioning.
Currently, Corporate Social Responsibility (CSR) initiatives by the private sector show promising results. Be it is the Tata Foundation supporting weavers in Varanasi or the Bill and Melinda Gates Foundation providing grants to fight Ebola in West Africa, the private sector is increasingly becoming part of a shared agenda to end global poverty. Private-public partnerships can also play a crucial role in elevating livelihoods. Recently, an IFAD-supported project in Rwanda forged a partnership between tea cooperatives and private investors- ensuring 30-40% equity shares for the cooperatives.
Further, private sector jobs are more diversified and generate higher wages. If higher incomes and job security is a marker of poverty alleviation, then the private sector is steadily contributing to reducing inequalities. It generates employment and makes livelihoods higher. In addition, there is immense potential for commercial credit to benefit the lives of the rural poor. While complete credit penetration into the rural hinterland and urban slums is yet to be seen, the availability of commercial credit will no doubt protect the poor against exorbitant interest rates and debt traps.
Concludingly, the private sector can end poverty. It can do so because of the new ideas it can bring to the table - innovation, strategy and skills. It has the immense resources it currently wields, and the humongous amount of influence that comes with it. The sector can participate because it is in its own interests to do so - in terms of opening up new markets and tapping new possibilities. All it needs is a reworking of the concept of ‘profit’ - from being a commercial end goal accruing to shareholders, to a social product to be distributed among all stakeholders.

 

Q. Why are private sector jobs a more desirable alternative for alleviating poverty?

Detailed Solution for CAT Practice Test - 26 - Question 5

Option 1 is mentioned in the passage “Further, private sector jobs are more diversified and generate higher wages.” It aptly provides an answer for the question presented.
Options 2 is a consequence of securing a private sector job. Option 3 is contextually misleading as the passage states that “there is immense potential for commercial credit to benefit the lives of the rural poor.” Option 4 can be eliminated.
Hence, the correct answer is option 1.

CAT Practice Test - 26 - Question 6

We live in a time when about 896 million people survive on less than $1.90 a day. A time, when the world produces sufficient food to provide 2700 k/cal to every person every day. Yet, a vast section of humanity is unable to afford it. The reason being this is the paucity of funds. Traditionally, the private sector has always been associated with the relentless drive to maximise profits, often at the cost of rising inequality. Whereas, the public sector is shown to pay greater heed to the distributive aspect of economic benefits. The private - public difference rests on the relative weights attached to efficiency and equity. While the private sector limits its arena of concern to the bottom line - subtraction of costs from revenues, the overarching role of the public sector is linked to enhancing welfare and promoting equity.
Poverty eradication has perks for the private sector too. Bottom of the Pyramid investment by multinational corporations, such as those pursued by Danone in India and Unilever in Africa, helps to lift billions out of poverty. This, in turn, allows deprived communities to access products and services that have been appropriately priced for them. Whatever the private sector loses in terms of margins, it gains in terms of volumes. This way, the private sector is able to tap into new markets and geographies. Further, participating in the development process allows the private sector to gain in terms of risk mitigation and greater value creation, in addition to positive social positioning.
Currently, Corporate Social Responsibility (CSR) initiatives by the private sector show promising results. Be it is the Tata Foundation supporting weavers in Varanasi or the Bill and Melinda Gates Foundation providing grants to fight Ebola in West Africa, the private sector is increasingly becoming part of a shared agenda to end global poverty. Private-public partnerships can also play a crucial role in elevating livelihoods. Recently, an IFAD-supported project in Rwanda forged a partnership between tea cooperatives and private investors- ensuring 30-40% equity shares for the cooperatives.
Further, private sector jobs are more diversified and generate higher wages. If higher incomes and job security is a marker of poverty alleviation, then the private sector is steadily contributing to reducing inequalities. It generates employment and makes livelihoods higher. In addition, there is immense potential for commercial credit to benefit the lives of the rural poor. While complete credit penetration into the rural hinterland and urban slums is yet to be seen, the availability of commercial credit will no doubt protect the poor against exorbitant interest rates and debt traps.
Concludingly, the private sector can end poverty. It can do so because of the new ideas it can bring to the table - innovation, strategy and skills. It has the immense resources it currently wields, and the humongous amount of influence that comes with it. The sector can participate because it is in its own interests to do so - in terms of opening up new markets and tapping new possibilities. All it needs is a reworking of the concept of ‘profit’ - from being a commercial end goal accruing to shareholders, to a social product to be distributed among all stakeholders.

 

Q. “All it needs is a reworking of the concept of ‘profit’ - from being a commercial end goal accruing to shareholders, to a social product to be distributed among all stakeholders.” implies that:

Detailed Solution for CAT Practice Test - 26 - Question 6

Option 1 is incorrect as “wrong definition” is contextually misleading and not the same as “reworking of the concept.” Option 2 can be inferred from the statement made.
Options 3 and 4 cannot be deduced from the passage.
Hence, the correct answer is option 2.

CAT Practice Test - 26 - Question 7

Group Question

A passage is followed by questions pertaining to the passage. Read the passage and answer the questions. Choose the most appropriate answer.

Countries often provide support for their farmers using trade barriers and subsidy because: domestic agriculture, even if it is inefficient by world standards, can be an insurance policy in case it becomes difficult (as it does, for example, in wartime) to buy agricultural produce from abroad; farmers groups have proved adept at lobbying; politicians have sought to slow the depopulation of rural areas; agricultural prices can be volatile, as a result of unpredictable weather, among other things; and financial support can provide a safety net in unexpectedly severe market conditions.
Broadly speaking, governments have tried two methods of subsidising agriculture. The first, used in the United States during the 1930s and in the UK before it joined the European Union, is to top up farmers' incomes if they fall below a level deemed acceptable. Farmers may be required to set aside some of their land in return for this support. The second is to guarantee a minimum level of farm prices by buying up surplus supply and storing or destroying it if prices would otherwise fall below the guaranteed levels. This was the approach adopted by the EU when it set up its Common Agricultural Policy. To keep down the direct cost of this subsidy the EU used trade barriers, including import levies, to minimise competition to EU farmers from produce available more cheaply on world agriculture markets. Recent American farm-support policy has combined income top-ups and some guaranteed prices. As most governments have become more committed to international trade, such agricultural policies have come under increasing attack, although the free trade rhetoric has often run far ahead of genuine reform. In 2003, rich countries together spent over $300 billion a year supporting their farmers, more than six times what they spent on foreign aid. Finding a way to end agricultural support had become by far the biggest remaining challenge for those trying to negotiate global free trade.

 

Q. Which of the following is the most suitable title for the above passage?

Detailed Solution for CAT Practice Test - 26 - Question 7

The first paragraph is about various reasons for supporting a country’s agricultural system. The second paragraph mentions different ways (practiced in US and UK) of how agriculture (farmers) is supported through income top-ups and guaranteed prices. The passage ends with the mention of agricultural subsidies being the biggest challenge for those in favour of global free trade.
Options 1, 3 and 4 do not mention agriculture, which is a key focus area in the passage. Therefore, they cannot be considered.
Hence, the correct answer is option 2.

CAT Practice Test - 26 - Question 8

Countries often provide support for their farmers using trade barriers and subsidy because: domestic agriculture, even if it is inefficient by world standards, can be an insurance policy in case it becomes difficult (as it does, for example, in wartime) to buy agricultural produce from abroad; farmers groups have proved adept at lobbying; politicians have sought to slow the depopulation of rural areas; agricultural prices can be volatile, as a result of unpredictable weather, among other things; and financial support can provide a safety net in unexpectedly severe market conditions.
Broadly speaking, governments have tried two methods of subsidising agriculture. The first, used in the United States during the 1930s and in the UK before it joined the European Union, is to top up farmers' incomes if they fall below a level deemed acceptable. Farmers may be required to set aside some of their land in return for this support. The second is to guarantee a minimum level of farm prices by buying up surplus supply and storing or destroying it if prices would otherwise fall below the guaranteed levels. This was the approach adopted by the EU when it set up its Common Agricultural Policy. To keep down the direct cost of this subsidy the EU used trade barriers, including import levies, to minimise competition to EU farmers from produce available more cheaply on world agriculture markets. Recent American farm-support policy has combined income top-ups and some guaranteed prices. As most governments have become more committed to international trade, such agricultural policies have come under increasing attack, although the free trade rhetoric has often run far ahead of genuine reform. In 2003, rich countries together spent over $300 billion a year supporting their farmers, more than six times what they spent on foreign aid. Finding a way to end agricultural support had become by far the biggest remaining challenge for those trying to negotiate global free trade.

 

Q. Trade barriers and subsidies as agricultural policies arise because of
A. globally competitive domestic agriculture.
B. political pressure from farmers and politicians.
C. market conditions.
D. unpredictable weather.

Detailed Solution for CAT Practice Test - 26 - Question 8

Statement A is incorrect because if the domestic agriculture is globally competitive in all respects, then trade barriers and subsidies are not required to protect it. Eliminate options 1 and 2.
Statements B, C and D have been mentioned in the passage almost verbatim as important reasons for trade barriers and subsidies with regard to agriculture.
Hence, the correct answer is option 3.

CAT Practice Test - 26 - Question 9

Countries often provide support for their farmers using trade barriers and subsidy because: domestic agriculture, even if it is inefficient by world standards, can be an insurance policy in case it becomes difficult (as it does, for example, in wartime) to buy agricultural produce from abroad; farmers groups have proved adept at lobbying; politicians have sought to slow the depopulation of rural areas; agricultural prices can be volatile, as a result of unpredictable weather, among other things; and financial support can provide a safety net in unexpectedly severe market conditions.
Broadly speaking, governments have tried two methods of subsidising agriculture. The first, used in the United States during the 1930s and in the UK before it joined the European Union, is to top up farmers' incomes if they fall below a level deemed acceptable. Farmers may be required to set aside some of their land in return for this support. The second is to guarantee a minimum level of farm prices by buying up surplus supply and storing or destroying it if prices would otherwise fall below the guaranteed levels. This was the approach adopted by the EU when it set up its Common Agricultural Policy. To keep down the direct cost of this subsidy the EU used trade barriers, including import levies, to minimise competition to EU farmers from produce available more cheaply on world agriculture markets. Recent American farm-support policy has combined income top-ups and some guaranteed prices. As most governments have become more committed to international trade, such agricultural policies have come under increasing attack, although the free trade rhetoric has often run far ahead of genuine reform. In 2003, rich countries together spent over $300 billion a year supporting their farmers, more than six times what they spent on foreign aid. Finding a way to end agricultural support had become by far the biggest remaining challenge for those trying to negotiate global free trade.

 

Q. According to the passage, governments subsidize agriculture in which of the following ways?
A. Governments directly pay the farmer.
B. Governments cut down on foreign aid.
C. Governments procure farm produce to maintain prices.
D. Governments impose import levies.

Detailed Solution for CAT Practice Test - 26 - Question 9

Governments cutting down on foreign aid is neither stated nor implied in the passage. The foreign aid is mentioned only in order to highlight the fact that governments spend more on protectionism than on foreign aid. Cutting down foreign aid is irrelevant. Therefore, statement B is incorrect.
This eliminates options 1, 2 and 4.
Statements A, C and D have all been mentioned in the passage.
Hence, the correct answer is option 3.

CAT Practice Test - 26 - Question 10

Group Question

A passage is followed by questions pertaining to the passage. Read the passage and answer the questions. Choose the most appropriate answer.

His first one-man art-gallery exhibition as a fine artist was on July 9, 1962, in the Ferus Gallery of Los Angeles. The exhibition marked the West Coast debut of pop art. Andy Warhol's first New York solo Pop exhibit was hosted at Eleanor Ward's Stable Gallery November 6-24, 1962. The exhibit included the works Marilyn Diptych, 100 Soup Cans, 100 Coke Bottles and 100 Dollar Bills. At the Stable Gallery exhibit, the artist met for the first time John Giorno who would star in Warhol's first film, Sleep, in 1963.
It was during the 1960s that Warhol began to make paintings of iconic American products such as Campbell's Soup Cans and Coca-Cola bottles, as well as paintings of celebrities such as Marilyn Monroe, Elvis Presley, Troy Donahue, Muhammad AN and Elizabeth Taylor. He founded "The Factory", his studio during these years, and gathered around himself a wide range of artists, writers, musicians, and underground celebrities. He began producing prints using the silkscreen method. His work became popular and controversial.

Among the imagery tackled by Warhol were dollar bills, celebrities and brand name products. He also used as imagery for his paintings newspaper headlines or photographs of mushroom clouds, electric chairs, and police dogs attacking civil rights protesters. Warhol also used Coca Cola bottles as subject matter for paintings. He had this to say about Coca Cola:“What's great about this country is that America started the tradition where the richest consumers buy essentially the same things as the poorest. You can be watching TV and see Coca Cola, and you know that the President drinks Coca Cola, Liz Taylor drinks Coca Cola, and just think, you can drink Coca Cola, too. A coke is a coke and no amount of money can get you a better coke than the one the bum on the corner is drinking. All the cokes are the same and all the cokes are good. Liz Taylor knows it, the President knows it, the bum knows it, and you know it.”

New York's Museum of Modern Art hosted a Symposium on pop art in December 1962 during which artists like Warhol were attacked for "capitulating" to consumerism. Critics were scandalized by Warhol's open embrace of market culture. This symposium set the tone for Warhol's reception. Throughout the decade it became more and more clear that there had been a profound change in the culture of the art world, and that Warhol was at the center of that shift.

 

Q. According to the passage, which of the following statements is Andy Warhol least likely to agree with?

Detailed Solution for CAT Practice Test - 26 - Question 10

All the passage offers in terms of information on pop art is that Andy Warhol was the perpetrator. “His first one-man art-gallery exhibition as a fine artist was on July 9, 1962, in the Ferus Gallery of Los Angeles. The exhibition marked the West Coast debut of pop art.” There is nothing to justify option 3.
Option 1 is eliminated as Warhol agrees with consumerism - “Critics were scandalized by Warhol's open embrace of market culture.” Option 2 is eliminated from “Warhol began to make paintings of celebrities such as Marilyn Monroe, Elvis Presley, Troy Donahue, Muhammad Ali and Elizabeth Taylor”.
Option 4 is eliminated Warhol talks about how anyone regardless of economic background can drink a Coke, and it will look, feel and taste the same.
Hence, the correct answer is option 3.

CAT Practice Test - 26 - Question 11

His first one-man art-gallery exhibition as a fine artist was on July 9, 1962, in the Ferus Gallery of Los Angeles. The exhibition marked the West Coast debut of pop art. Andy Warhol's first New York solo Pop exhibit was hosted at Eleanor Ward's Stable Gallery November 6-24, 1962. The exhibit included the works Marilyn Diptych, 100 Soup Cans, 100 Coke Bottles and 100 Dollar Bills. At the Stable Gallery exhibit, the artist met for the first time John Giorno who would star in Warhol's first film, Sleep, in 1963.
It was during the 1960s that Warhol began to make paintings of iconic American products such as Campbell's Soup Cans and Coca-Cola bottles, as well as paintings of celebrities such as Marilyn Monroe, Elvis Presley, Troy Donahue, Muhammad AN and Elizabeth Taylor. He founded "The Factory", his studio during these years, and gathered around himself a wide range of artists, writers, musicians, and underground celebrities. He began producing prints using the silkscreen method. His work became popular and controversial.

Among the imagery tackled by Warhol were dollar bills, celebrities and brand name products. He also used as imagery for his paintings newspaper headlines or photographs of mushroom clouds, electric chairs, and police dogs attacking civil rights protesters. Warhol also used Coca Cola bottles as subject matter for paintings. He had this to say about Coca Cola:“What's great about this country is that America started the tradition where the richest consumers buy essentially the same things as the poorest. You can be watching TV and see Coca Cola, and you know that the President drinks Coca Cola, Liz Taylor drinks Coca Cola, and just think, you can drink Coca Cola, too. A coke is a coke and no amount of money can get you a better coke than the one the bum on the corner is drinking. All the cokes are the same and all the cokes are good. Liz Taylor knows it, the President knows it, the bum knows it, and you know it.”

New York's Museum of Modern Art hosted a Symposium on pop art in December 1962 during which artists like Warhol were attacked for "capitulating" to consumerism. Critics were scandalized by Warhol's open embrace of market culture. This symposium set the tone for Warhol's reception. Throughout the decade it became more and more clear that there had been a profound change in the culture of the art world, and that Warhol was at the center of that shift.

 

Q. Which of the following words come closest in meaning to the word “capitulate”?

Detailed Solution for CAT Practice Test - 26 - Question 11

The word capitulate means “to surrender unconditionally or on stipulated terms.” The word that comes closest in meaning to it is defer, because it means “to yield respectfully in judgment or opinion.” Seize means “to take hold of suddenly or forcibly; grasp” Attack means “to set upon in a forceful, violent, hostile, or aggressive way” Acquire means “to gain for oneself through one's actions or efforts” Hence, the correct answer is option 1.

CAT Practice Test - 26 - Question 12

His first one-man art-gallery exhibition as a fine artist was on July 9, 1962, in the Ferus Gallery of Los Angeles. The exhibition marked the West Coast debut of pop art. Andy Warhol's first New York solo Pop exhibit was hosted at Eleanor Ward's Stable Gallery November 6-24, 1962. The exhibit included the works Marilyn Diptych, 100 Soup Cans, 100 Coke Bottles and 100 Dollar Bills. At the Stable Gallery exhibit, the artist met for the first time John Giorno who would star in Warhol's first film, Sleep, in 1963.
It was during the 1960s that Warhol began to make paintings of iconic American products such as Campbell's Soup Cans and Coca-Cola bottles, as well as paintings of celebrities such as Marilyn Monroe, Elvis Presley, Troy Donahue, Muhammad AN and Elizabeth Taylor. He founded "The Factory", his studio during these years, and gathered around himself a wide range of artists, writers, musicians, and underground celebrities. He began producing prints using the silkscreen method. His work became popular and controversial.

Among the imagery tackled by Warhol were dollar bills, celebrities and brand name products. He also used as imagery for his paintings newspaper headlines or photographs of mushroom clouds, electric chairs, and police dogs attacking civil rights protesters. Warhol also used Coca Cola bottles as subject matter for paintings. He had this to say about Coca Cola:“What's great about this country is that America started the tradition where the richest consumers buy essentially the same things as the poorest. You can be watching TV and see Coca Cola, and you know that the President drinks Coca Cola, Liz Taylor drinks Coca Cola, and just think, you can drink Coca Cola, too. A coke is a coke and no amount of money can get you a better coke than the one the bum on the corner is drinking. All the cokes are the same and all the cokes are good. Liz Taylor knows it, the President knows it, the bum knows it, and you know it.”

New York's Museum of Modern Art hosted a Symposium on pop art in December 1962 during which artists like Warhol were attacked for "capitulating" to consumerism. Critics were scandalized by Warhol's open embrace of market culture. This symposium set the tone for Warhol's reception. Throughout the decade it became more and more clear that there had been a profound change in the culture of the art world, and that Warhol was at the center of that shift.

 

Q. Which of the following art forms did Warhol not dabble in?

Detailed Solution for CAT Practice Test - 26 - Question 12

Warhol didn’t dabble in music - he was just surrounded by musicians. “He founded "The Factory", his studio during these years, and gathered around himself a wide range of artists, writers, musicians, and underground celebrities.” We know he did painting - “His first one-man art-gallery exhibition as a fine artist was on July 9, 1962, in the Ferus Gallery of Los Angeles.” Therefore Option 1 is eliminated.
We know he did screen printing - “He began producing prints using the silkscreen method.” We know he dabbled in film-making also “At the Stable Gallery exhibit, the artist met for the first time John Giorno who would star in Warhol's first film, Sleep, in 1963.” Hence, the correct answer is option 4.

CAT Practice Test - 26 - Question 13

Group Question

A passage is followed by questions pertaining to the passage. Read the passage and answer the questions. Choose the most appropriate answer.

Since the publication of The Lord of the Rings by J. R. R. Tolkien, a wealth of secondary literature has been published discussing the literary themes and archetypes present in the story. Tolkien also wrote about the themes of his book in letters to friends, family and fans, and also in the book itself. In his Foreword to the Second Edition, Tolkien said that he "disliked allegory in all its forms" (using the word applicability instead), and told those claiming the story was a metaphor for World War II to remember that he had lost "all but one" of his close friends in World War I. "No careful reader of Tolkien's fiction can fail to be aware of the polarities that give it form and fiction," writes Verlyn Flieger. Tolkien's extensive use of duality and parallelism, contrast and opposition is found throughout the novel, in hope and despair, ignorance and enlightenment, death and immortality, fate and free will. One famous example is the often criticized polarity between Evil and Good in Tolkien. Ores, the most maligned of races, are a corruption of the mystically exalted race of the Elves. Minas Morgul, the Tower of Sorcery, home of the Lord of the NazgOI, the most corrupted Kings of Men, directly opposes Minas Tirith, the Tower of Guard and the capital of Gondor, the last visible remnant of the ancient kingdom of Men in the Third Age.
The antitheses, though pronounced and prolific, are sometimes seen to be too polarizing, but they have also been argued to be at the heart of the structure of the entire story. Tolkien's technique has been seen to "confer literality on what would in the primary world be called metaphor and then to illustrate [in his secondary world] the process by which the literal becomes metaphoric." A famous description of this device is Verlyn Flieger's Splintered Light where the mythology of the Elves described in The Silmarillion is seen not only to be the story of the fall of the Elves from grace (a Fall akin to the Fall of Satan or Adam and Eve) due to the hubris of Feanor in his deadly oath regarding the Silmarils and all that follows as a result of it, but also a story built on a simultaneous splintering of light from the light of creation and the splintering of Elvish language from the word of creation, Ea. Although, these arguments are more readily seen in The Silmarillion, which contains the Creation Myth of the Elves, similar observations can and have also been made regarding The Lord of the Rings.
The theme of power in The Lord of the Rings centres around the corrupting influence of the One Ring. This theme is discussed at length by Tom Shippey in chapter III of J. R. R. Tolkien: Author of the Century. In this chapter, titled, "The Lord of the Rings: Concepts of Evil", Shippey notes that what lies at the heart of the story is the assertions made by Gandalf about the power and influence of the One Ring, and the corrupting influence it has on its bearers. Gandalf rejects the Ring after Frodo offers it to him, and this view of the nature of the Ring is reinforced as Elrond and Galadriel in their turn, also reject the Ring.
This is, according to Shippey, a very modern, 20th-century theme, since in earlier, medieval times, power was considered to "reveal character", not alter it. Shippey mentions Lord Acton's famous statement in 1887, that "Power tends to corrupt, and absolute power corrupts absolutely. Great men are almost always bad men..." He then goes on to point out authors that were dealing in the same themes of power and corruption at around the same time as Tolkien wrote his work. These authors include George Orwell with Animal Farm (1945), William Golding with The Lord of the Flies (1954) and The Inheritors (1955), and T. H. White with The Once and Future King (1958).

 

Q. What does Tolkien have to say to people who claim that Lord of the Rings was metaphor for World War II?

Detailed Solution for CAT Practice Test - 26 - Question 13

The passage states that Tolkien “told those claiming the story was a metaphor for World War II to remember that he had lost "all but one" of his close friends in World War I.” Which clearly meant to serve as a reminder for his probable resulting dislike for war.
Option 1 is broad and vague with 'association'.
Option 3 is disconnected from the question stem.
Option 4 is wrong as the passage doesn’t convey his “hatred” - just that he might not like war.
Hence, the correct answer is option 2.

CAT Practice Test - 26 - Question 14

Since the publication of The Lord of the Rings by J. R. R. Tolkien, a wealth of secondary literature has been published discussing the literary themes and archetypes present in the story. Tolkien also wrote about the themes of his book in letters to friends, family and fans, and also in the book itself. In his Foreword to the Second Edition, Tolkien said that he "disliked allegory in all its forms" (using the word applicability instead), and told those claiming the story was a metaphor for World War II to remember that he had lost "all but one" of his close friends in World War I. "No careful reader of Tolkien's fiction can fail to be aware of the polarities that give it form and fiction," writes Verlyn Flieger. Tolkien's extensive use of duality and parallelism, contrast and opposition is found throughout the novel, in hope and despair, ignorance and enlightenment, death and immortality, fate and free will. One famous example is the often criticized polarity between Evil and Good in Tolkien. Ores, the most maligned of races, are a corruption of the mystically exalted race of the Elves. Minas Morgul, the Tower of Sorcery, home of the Lord of the NazgOI, the most corrupted Kings of Men, directly opposes Minas Tirith, the Tower of Guard and the capital of Gondor, the last visible remnant of the ancient kingdom of Men in the Third Age.
The antitheses, though pronounced and prolific, are sometimes seen to be too polarizing, but they have also been argued to be at the heart of the structure of the entire story. Tolkien's technique has been seen to "confer literality on what would in the primary world be called metaphor and then to illustrate [in his secondary world] the process by which the literal becomes metaphoric." A famous description of this device is Verlyn Flieger's Splintered Light where the mythology of the Elves described in The Silmarillion is seen not only to be the story of the fall of the Elves from grace (a Fall akin to the Fall of Satan or Adam and Eve) due to the hubris of Feanor in his deadly oath regarding the Silmarils and all that follows as a result of it, but also a story built on a simultaneous splintering of light from the light of creation and the splintering of Elvish language from the word of creation, Ea. Although, these arguments are more readily seen in The Silmarillion, which contains the Creation Myth of the Elves, similar observations can and have also been made regarding The Lord of the Rings.
The theme of power in The Lord of the Rings centres around the corrupting influence of the One Ring. This theme is discussed at length by Tom Shippey in chapter III of J. R. R. Tolkien: Author of the Century. In this chapter, titled, "The Lord of the Rings: Concepts of Evil", Shippey notes that what lies at the heart of the story is the assertions made by Gandalf about the power and influence of the One Ring, and the corrupting influence it has on its bearers. Gandalf rejects the Ring after Frodo offers it to him, and this view of the nature of the Ring is reinforced as Elrond and Galadriel in their turn, also reject the Ring.
This is, according to Shippey, a very modern, 20th-century theme, since in earlier, medieval times, power was considered to "reveal character", not alter it. Shippey mentions Lord Acton's famous statement in 1887, that "Power tends to corrupt, and absolute power corrupts absolutely. Great men are almost always bad men..." He then goes on to point out authors that were dealing in the same themes of power and corruption at around the same time as Tolkien wrote his work. These authors include George Orwell with Animal Farm (1945), William Golding with The Lord of the Flies (1954) and The Inheritors (1955), and T. H. White with The Once and Future King (1958).

 

Q. Which of the following aren’t paradoxes found throughout Lord of the Rings?

Detailed Solution for CAT Practice Test - 26 - Question 14

A paradox is something that holds two opposing ideas within itself. The passage states that “Tolkien's extensive use of duality and parallelism, contrast and opposition is found throughout the novel, in hope and despair, ignorance and enlightenment, death and immortality, fate and free will.” Eliminate options 1 and 2.
Option 3 is eliminated as it is also mentioned as a notable polarity - “One famous example is the often criticized polarity between Evil and Good in Tolkien.” Now, Metaphors and Similes essentially compare two entities and there is no paradox presented in them (by Tolkien).
Hence, the correct answer is option 4.

CAT Practice Test - 26 - Question 15

Since the publication of The Lord of the Rings by J. R. R. Tolkien, a wealth of secondary literature has been published discussing the literary themes and archetypes present in the story. Tolkien also wrote about the themes of his book in letters to friends, family and fans, and also in the book itself. In his Foreword to the Second Edition, Tolkien said that he "disliked allegory in all its forms" (using the word applicability instead), and told those claiming the story was a metaphor for World War II to remember that he had lost "all but one" of his close friends in World War I. "No careful reader of Tolkien's fiction can fail to be aware of the polarities that give it form and fiction," writes Verlyn Flieger. Tolkien's extensive use of duality and parallelism, contrast and opposition is found throughout the novel, in hope and despair, ignorance and enlightenment, death and immortality, fate and free will. One famous example is the often criticized polarity between Evil and Good in Tolkien. Ores, the most maligned of races, are a corruption of the mystically exalted race of the Elves. Minas Morgul, the Tower of Sorcery, home of the Lord of the NazgOI, the most corrupted Kings of Men, directly opposes Minas Tirith, the Tower of Guard and the capital of Gondor, the last visible remnant of the ancient kingdom of Men in the Third Age.
The antitheses, though pronounced and prolific, are sometimes seen to be too polarizing, but they have also been argued to be at the heart of the structure of the entire story. Tolkien's technique has been seen to "confer literality on what would in the primary world be called metaphor and then to illustrate [in his secondary world] the process by which the literal becomes metaphoric." A famous description of this device is Verlyn Flieger's Splintered Light where the mythology of the Elves described in The Silmarillion is seen not only to be the story of the fall of the Elves from grace (a Fall akin to the Fall of Satan or Adam and Eve) due to the hubris of Feanor in his deadly oath regarding the Silmarils and all that follows as a result of it, but also a story built on a simultaneous splintering of light from the light of creation and the splintering of Elvish language from the word of creation, Ea. Although, these arguments are more readily seen in The Silmarillion, which contains the Creation Myth of the Elves, similar observations can and have also been made regarding The Lord of the Rings.
The theme of power in The Lord of the Rings centres around the corrupting influence of the One Ring. This theme is discussed at length by Tom Shippey in chapter III of J. R. R. Tolkien: Author of the Century. In this chapter, titled, "The Lord of the Rings: Concepts of Evil", Shippey notes that what lies at the heart of the story is the assertions made by Gandalf about the power and influence of the One Ring, and the corrupting influence it has on its bearers. Gandalf rejects the Ring after Frodo offers it to him, and this view of the nature of the Ring is reinforced as Elrond and Galadriel in their turn, also reject the Ring.
This is, according to Shippey, a very modern, 20th-century theme, since in earlier, medieval times, power was considered to "reveal character", not alter it. Shippey mentions Lord Acton's famous statement in 1887, that "Power tends to corrupt, and absolute power corrupts absolutely. Great men are almost always bad men..." He then goes on to point out authors that were dealing in the same themes of power and corruption at around the same time as Tolkien wrote his work. These authors include George Orwell with Animal Farm (1945), William Golding with The Lord of the Flies (1954) and The Inheritors (1955), and T. H. White with The Once and Future King (1958).

 

Q. Which of the following can’t be inferred about the Elves in this passage?

Detailed Solution for CAT Practice Test - 26 - Question 15

The passage acknowledges that the elves are an exalted race - “One famous example is the often criticized polarity between Evil and Good in Tolkien. Ores, the most maligned of races, are a corruption of the mystically exalted race of the Elves.” The word “exalted” is a synonym to the word glorious- eliminate option 1.
Option 2 can be eliminated as it was Feanor’s hubris - another word for arrogance, that leads to the Elves’ fall from grace. Option 4 can be eliminated as “Although, these arguments are more readily seen in The Silmarillion, which contains the Creation Myth of the Elves” There is no proof in the passage to support option 3.
Hence, the correct answer is option 3.

CAT Practice Test - 26 - Question 16

Since the publication of The Lord of the Rings by J. R. R. Tolkien, a wealth of secondary literature has been published discussing the literary themes and archetypes present in the story. Tolkien also wrote about the themes of his book in letters to friends, family and fans, and also in the book itself. In his Foreword to the Second Edition, Tolkien said that he "disliked allegory in all its forms" (using the word applicability instead), and told those claiming the story was a metaphor for World War II to remember that he had lost "all but one" of his close friends in World War I. "No careful reader of Tolkien's fiction can fail to be aware of the polarities that give it form and fiction," writes Verlyn Flieger. Tolkien's extensive use of duality and parallelism, contrast and opposition is found throughout the novel, in hope and despair, ignorance and enlightenment, death and immortality, fate and free will. One famous example is the often criticized polarity between Evil and Good in Tolkien. Ores, the most maligned of races, are a corruption of the mystically exalted race of the Elves. Minas Morgul, the Tower of Sorcery, home of the Lord of the NazgOI, the most corrupted Kings of Men, directly opposes Minas Tirith, the Tower of Guard and the capital of Gondor, the last visible remnant of the ancient kingdom of Men in the Third Age.
The antitheses, though pronounced and prolific, are sometimes seen to be too polarizing, but they have also been argued to be at the heart of the structure of the entire story. Tolkien's technique has been seen to "confer literality on what would in the primary world be called metaphor and then to illustrate [in his secondary world] the process by which the literal becomes metaphoric." A famous description of this device is Verlyn Flieger's Splintered Light where the mythology of the Elves described in The Silmarillion is seen not only to be the story of the fall of the Elves from grace (a Fall akin to the Fall of Satan or Adam and Eve) due to the hubris of Feanor in his deadly oath regarding the Silmarils and all that follows as a result of it, but also a story built on a simultaneous splintering of light from the light of creation and the splintering of Elvish language from the word of creation, Ea. Although, these arguments are more readily seen in The Silmarillion, which contains the Creation Myth of the Elves, similar observations can and have also been made regarding The Lord of the Rings.
The theme of power in The Lord of the Rings centres around the corrupting influence of the One Ring. This theme is discussed at length by Tom Shippey in chapter III of J. R. R. Tolkien: Author of the Century. In this chapter, titled, "The Lord of the Rings: Concepts of Evil", Shippey notes that what lies at the heart of the story is the assertions made by Gandalf about the power and influence of the One Ring, and the corrupting influence it has on its bearers. Gandalf rejects the Ring after Frodo offers it to him, and this view of the nature of the Ring is reinforced as Elrond and Galadriel in their turn, also reject the Ring.
This is, according to Shippey, a very modern, 20th-century theme, since in earlier, medieval times, power was considered to "reveal character", not alter it. Shippey mentions Lord Acton's famous statement in 1887, that "Power tends to corrupt, and absolute power corrupts absolutely. Great men are almost always bad men..." He then goes on to point out authors that were dealing in the same themes of power and corruption at around the same time as Tolkien wrote his work. These authors include George Orwell with Animal Farm (1945), William Golding with The Lord of the Flies (1954) and The Inheritors (1955), and T. H. White with The Once and Future King (1958).

 

Q. What do J.R.R Tolkien and T.H.White not have in common?

Detailed Solution for CAT Practice Test - 26 - Question 16

The passage states that - “He (Shippey) then goes on to point out authors that were dealing in the same themes of power and corruption at around the same time as Tolkien wrote his work. These authors include George Orwell with Animal Farm (1945), William Golding with The Lord of the Flies (1954) and The Inheritors (1955), and T. H. White with The Once and Future King (1958).”
Thus we can conclude that they existed around the same time, and that they were about power and corruption, they were novelists, and that Shippey mentions them.
Nowhere does it say that T.H. White’s The Once and Future King is a fantasy novel.
Hence, the correct answer is option 4.

CAT Practice Test - 26 - Question 17

Since the publication of The Lord of the Rings by J. R. R. Tolkien, a wealth of secondary literature has been published discussing the literary themes and archetypes present in the story. Tolkien also wrote about the themes of his book in letters to friends, family and fans, and also in the book itself. In his Foreword to the Second Edition, Tolkien said that he "disliked allegory in all its forms" (using the word applicability instead), and told those claiming the story was a metaphor for World War II to remember that he had lost "all but one" of his close friends in World War I. "No careful reader of Tolkien's fiction can fail to be aware of the polarities that give it form and fiction," writes Verlyn Flieger. Tolkien's extensive use of duality and parallelism, contrast and opposition is found throughout the novel, in hope and despair, ignorance and enlightenment, death and immortality, fate and free will. One famous example is the often criticized polarity between Evil and Good in Tolkien. Ores, the most maligned of races, are a corruption of the mystically exalted race of the Elves. Minas Morgul, the Tower of Sorcery, home of the Lord of the NazgOI, the most corrupted Kings of Men, directly opposes Minas Tirith, the Tower of Guard and the capital of Gondor, the last visible remnant of the ancient kingdom of Men in the Third Age.
The antitheses, though pronounced and prolific, are sometimes seen to be too polarizing, but they have also been argued to be at the heart of the structure of the entire story. Tolkien's technique has been seen to "confer literality on what would in the primary world be called metaphor and then to illustrate [in his secondary world] the process by which the literal becomes metaphoric." A famous description of this device is Verlyn Flieger's Splintered Light where the mythology of the Elves described in The Silmarillion is seen not only to be the story of the fall of the Elves from grace (a Fall akin to the Fall of Satan or Adam and Eve) due to the hubris of Feanor in his deadly oath regarding the Silmarils and all that follows as a result of it, but also a story built on a simultaneous splintering of light from the light of creation and the splintering of Elvish language from the word of creation, Ea. Although, these arguments are more readily seen in The Silmarillion, which contains the Creation Myth of the Elves, similar observations can and have also been made regarding The Lord of the Rings.
The theme of power in The Lord of the Rings centres around the corrupting influence of the One Ring. This theme is discussed at length by Tom Shippey in chapter III of J. R. R. Tolkien: Author of the Century. In this chapter, titled, "The Lord of the Rings: Concepts of Evil", Shippey notes that what lies at the heart of the story is the assertions made by Gandalf about the power and influence of the One Ring, and the corrupting influence it has on its bearers. Gandalf rejects the Ring after Frodo offers it to him, and this view of the nature of the Ring is reinforced as Elrond and Galadriel in their turn, also reject the Ring.
This is, according to Shippey, a very modern, 20th-century theme, since in earlier, medieval times, power was considered to "reveal character", not alter it. Shippey mentions Lord Acton's famous statement in 1887, that "Power tends to corrupt, and absolute power corrupts absolutely. Great men are almost always bad men..." He then goes on to point out authors that were dealing in the same themes of power and corruption at around the same time as Tolkien wrote his work. These authors include George Orwell with Animal Farm (1945), William Golding with The Lord of the Flies (1954) and The Inheritors (1955), and T. H. White with The Once and Future King (1958).

 

Q. Which of the following options are an apt title for the passage?

Detailed Solution for CAT Practice Test - 26 - Question 17

The passage is about the major recurring themes in the passage; the corruption of power and the polarities of the novel. Option 2 ignores “power”.
Option 3 is broad based.
Option 4 is too narrow with focus only on evil.
Only option 1 mentions the major recurring themes in the passage; the corruption of power and the polarities of the novel. Hence, the correct answer is option 1.

CAT Practice Test - 26 - Question 18

Since the publication of The Lord of the Rings by J. R. R. Tolkien, a wealth of secondary literature has been published discussing the literary themes and archetypes present in the story. Tolkien also wrote about the themes of his book in letters to friends, family and fans, and also in the book itself. In his Foreword to the Second Edition, Tolkien said that he "disliked allegory in all its forms" (using the word applicability instead), and told those claiming the story was a metaphor for World War II to remember that he had lost "all but one" of his close friends in World War I. "No careful reader of Tolkien's fiction can fail to be aware of the polarities that give it form and fiction," writes Verlyn Flieger. Tolkien's extensive use of duality and parallelism, contrast and opposition is found throughout the novel, in hope and despair, ignorance and enlightenment, death and immortality, fate and free will. One famous example is the often criticized polarity between Evil and Good in Tolkien. Ores, the most maligned of races, are a corruption of the mystically exalted race of the Elves. Minas Morgul, the Tower of Sorcery, home of the Lord of the NazgOI, the most corrupted Kings of Men, directly opposes Minas Tirith, the Tower of Guard and the capital of Gondor, the last visible remnant of the ancient kingdom of Men in the Third Age.
The antitheses, though pronounced and prolific, are sometimes seen to be too polarizing, but they have also been argued to be at the heart of the structure of the entire story. Tolkien's technique has been seen to "confer literality on what would in the primary world be called metaphor and then to illustrate [in his secondary world] the process by which the literal becomes metaphoric." A famous description of this device is Verlyn Flieger's Splintered Light where the mythology of the Elves described in The Silmarillion is seen not only to be the story of the fall of the Elves from grace (a Fall akin to the Fall of Satan or Adam and Eve) due to the hubris of Feanor in his deadly oath regarding the Silmarils and all that follows as a result of it, but also a story built on a simultaneous splintering of light from the light of creation and the splintering of Elvish language from the word of creation, Ea. Although, these arguments are more readily seen in The Silmarillion, which contains the Creation Myth of the Elves, similar observations can and have also been made regarding The Lord of the Rings.
The theme of power in The Lord of the Rings centres around the corrupting influence of the One Ring. This theme is discussed at length by Tom Shippey in chapter III of J. R. R. Tolkien: Author of the Century. In this chapter, titled, "The Lord of the Rings: Concepts of Evil", Shippey notes that what lies at the heart of the story is the assertions made by Gandalf about the power and influence of the One Ring, and the corrupting influence it has on its bearers. Gandalf rejects the Ring after Frodo offers it to him, and this view of the nature of the Ring is reinforced as Elrond and Galadriel in their turn, also reject the Ring.
This is, according to Shippey, a very modern, 20th-century theme, since in earlier, medieval times, power was considered to "reveal character", not alter it. Shippey mentions Lord Acton's famous statement in 1887, that "Power tends to corrupt, and absolute power corrupts absolutely. Great men are almost always bad men..." He then goes on to point out authors that were dealing in the same themes of power and corruption at around the same time as Tolkien wrote his work. These authors include George Orwell with Animal Farm (1945), William Golding with The Lord of the Flies (1954) and The Inheritors (1955), and T. H. White with The Once and Future King (1958).

 

Q. What can be said about Verlyn Flieger's Splintered Light?

Detailed Solution for CAT Practice Test - 26 - Question 18

The sentence, "A famous description....of creation, Ea." suggests that it is an analysis of Tolkein's “The Silmarillion” and the sentence “Although, ....The Lord of the Rings.” suggests the same about “Lord of the Rings”. This vindicates option 1 as the correct answer.
Options 2, 3 and 4 are incorrect as per passage.
Hence, the correct answer is option 1.

CAT Practice Test - 26 - Question 19

Group Question

For each of the passages below, read the passage, and mark the most appropriate answer to the question(s).

Structuration theory aims to avoid extremes of structural or agent determinism. The balancing of agency and structure is referred to as the duality of structure: social structures make social action possible and at the same time, that social action creates those very structures. For Giddens, structures are rules and resources (sets of transformation relations) organized as properties of social systems. Rules are patterns people may follow in social life. Resources relate to what is created by human action; they are not given by nature (explained further below). The theory employs a recursive notion of actions constrained and enabled by structures which are produced and reproduced by those actions. Consequently, this theory has been adopted by those with structuralist inclinations, but who wish to situate such structures in human practice rather than reify them as an ideal type or material property. (This is different, for example, from actor-network theory which grants certain autonomy to technical artefacts.) Additionally, the theory of structuration distinguishes between discursive and practical knowledge recognizes actors as having knowledge is reflexive and situated, and that habitual use becomes institutionalized.
A social system can be understood by its structure, modality, and interaction. Structure is constituted by rules and resources governing and available to agents. (Authoritative resources control persons, whereas allocative resources control material objects.) The modality of a structural system is the means by which structures are translated into action. Interaction is the activity instantiated by the agent acting within the social system. There has been some attempt by various theorists to link structuration theory to systems theory (with its emphasis on recursive loops) or the complexity theory of organizational structure (which emphasizes the adaptability that simple structures provide). Social systems have patterns of social relation that exist over time; the changing nature of space and time will determine the interaction of social relations and therefore structure. For example, 19th century Britain set out certain rules for that time and space. Those rules affected the action which determines structure and the structure was upheld as long as it was reproduced in action. Hitherto social structures or ‘models of society’ were taken to be beyond the realm of human control - the positivistic approach; the other social theory would be that of action creating society - the interpretivist approach. The duality of structure would argue that, in the most basic assumption, that they are one and the same - different sides to the coin of a similar problem of order.
Agency, as Giddens calls it, is human action. To be human is to be an agent, although not all agents are human beings. Agents’ knowledge of their society informs their action, which reproduce social structures, which in turn enforce and maintain the dynamics of action. Giddens defines ‘ontological security’ as the trust people have in social structure; everyday actions have some degree of predictability, thus ensuring social stability. This is not always true, though, as the possession of agency allows one to break away from normative actions, and depending on the sum of social factors at work, they may instigate shifts in the social structure. The dynamic between agency and structure makes such generative action possible. Thus agency can lead to both the reproduction and the transformation of society. Another way to explain this concept is by, what Giddens calls, the “reflexive monitoring of actions”. Reflexive monitoring looks at the ability to look at actions to judge their effectiveness in achieving their objectives: if agents can reproduce structure through action, they can also transform it.

 

Q. A structure’s modality is the basis for:

Detailed Solution for CAT Practice Test - 26 - Question 19

Option 1 is incorrect as the author mentions that social pattern is related to its time period and space. However it does not specify the modality of the structure as the basis of its relation. Option 2 like option 1 is incorrect as it too does not specify the modality of the structure as the basis of its interrelation with action.
The dynamism between structure and action makes is responsible for making generative transformation possible which makes option 4 incorrect.
The author clearly mentions that the modality of the structure is the means by which structures are translated into action. Hence, the correct answer is option 3.

CAT Practice Test - 26 - Question 20

Structuration theory aims to avoid extremes of structural or agent determinism. The balancing of agency and structure is referred to as the duality of structure: social structures make social action possible and at the same time, that social action creates those very structures. For Giddens, structures are rules and resources (sets of transformation relations) organized as properties of social systems. Rules are patterns people may follow in social life. Resources relate to what is created by human action; they are not given by nature (explained further below). The theory employs a recursive notion of actions constrained and enabled by structures which are produced and reproduced by those actions. Consequently, this theory has been adopted by those with structuralist inclinations, but who wish to situate such structures in human practice rather than reify them as an ideal type or material property. (This is different, for example, from actor-network theory which grants certain autonomy to technical artefacts.) Additionally, the theory of structuration distinguishes between discursive and practical knowledge recognizes actors as having knowledge is reflexive and situated, and that habitual use becomes institutionalized.
A social system can be understood by its structure, modality, and interaction. Structure is constituted by rules and resources governing and available to agents. (Authoritative resources control persons, whereas allocative resources control material objects.) The modality of a structural system is the means by which structures are translated into action. Interaction is the activity instantiated by the agent acting within the social system. There has been some attempt by various theorists to link structuration theory to systems theory (with its emphasis on recursive loops) or the complexity theory of organizational structure (which emphasizes the adaptability that simple structures provide). Social systems have patterns of social relation that exist over time; the changing nature of space and time will determine the interaction of social relations and therefore structure. For example, 19th century Britain set out certain rules for that time and space. Those rules affected the action which determines structure and the structure was upheld as long as it was reproduced in action. Hitherto social structures or ‘models of society’ were taken to be beyond the realm of human control - the positivistic approach; the other social theory would be that of action creating society - the interpretivist approach. The duality of structure would argue that, in the most basic assumption, that they are one and the same - different sides to the coin of a similar problem of order.
Agency, as Giddens calls it, is human action. To be human is to be an agent, although not all agents are human beings. Agents’ knowledge of their society informs their action, which reproduce social structures, which in turn enforce and maintain the dynamics of action. Giddens defines ‘ontological security’ as the trust people have in social structure; everyday actions have some degree of predictability, thus ensuring social stability. This is not always true, though, as the possession of agency allows one to break away from normative actions, and depending on the sum of social factors at work, they may instigate shifts in the social structure. The dynamic between agency and structure makes such generative action possible. Thus agency can lead to both the reproduction and the transformation of society. Another way to explain this concept is by, what Giddens calls, the “reflexive monitoring of actions”. Reflexive monitoring looks at the ability to look at actions to judge their effectiveness in achieving their objectives: if agents can reproduce structure through action, they can also transform it.

 

Q. The author presents an example of the 19th century in order to:

Detailed Solution for CAT Practice Test - 26 - Question 20

Options 1 and 2 are incorrect as they compare time and space and such a comparison is not mentioned in the passage. Option 4 mentions ‘modern society’- not indicated in the passage.
The passage states “Social systems have patterns of social relation that exist over time; the changing nature of space and time will determine the interaction of social relations and therefore structure”. This idea is captured in option 3.
Hence, the correct answer is option 3.

CAT Practice Test - 26 - Question 21

Structuration theory aims to avoid extremes of structural or agent determinism. The balancing of agency and structure is referred to as the duality of structure: social structures make social action possible and at the same time, that social action creates those very structures. For Giddens, structures are rules and resources (sets of transformation relations) organized as properties of social systems. Rules are patterns people may follow in social life. Resources relate to what is created by human action; they are not given by nature (explained further below). The theory employs a recursive notion of actions constrained and enabled by structures which are produced and reproduced by those actions. Consequently, this theory has been adopted by those with structuralist inclinations, but who wish to situate such structures in human practice rather than reify them as an ideal type or material property. (This is different, for example, from actor-network theory which grants certain autonomy to technical artefacts.) Additionally, the theory of structuration distinguishes between discursive and practical knowledge recognizes actors as having knowledge is reflexive and situated, and that habitual use becomes institutionalized.
A social system can be understood by its structure, modality, and interaction. Structure is constituted by rules and resources governing and available to agents. (Authoritative resources control persons, whereas allocative resources control material objects.) The modality of a structural system is the means by which structures are translated into action. Interaction is the activity instantiated by the agent acting within the social system. There has been some attempt by various theorists to link structuration theory to systems theory (with its emphasis on recursive loops) or the complexity theory of organizational structure (which emphasizes the adaptability that simple structures provide). Social systems have patterns of social relation that exist over time; the changing nature of space and time will determine the interaction of social relations and therefore structure. For example, 19th century Britain set out certain rules for that time and space. Those rules affected the action which determines structure and the structure was upheld as long as it was reproduced in action. Hitherto social structures or ‘models of society’ were taken to be beyond the realm of human control - the positivistic approach; the other social theory would be that of action creating society - the interpretivist approach. The duality of structure would argue that, in the most basic assumption, that they are one and the same - different sides to the coin of a similar problem of order.
Agency, as Giddens calls it, is human action. To be human is to be an agent, although not all agents are human beings. Agents’ knowledge of their society informs their action, which reproduce social structures, which in turn enforce and maintain the dynamics of action. Giddens defines ‘ontological security’ as the trust people have in social structure; everyday actions have some degree of predictability, thus ensuring social stability. This is not always true, though, as the possession of agency allows one to break away from normative actions, and depending on the sum of social factors at work, they may instigate shifts in the social structure. The dynamic between agency and structure makes such generative action possible. Thus agency can lead to both the reproduction and the transformation of society. Another way to explain this concept is by, what Giddens calls, the “reflexive monitoring of actions”. Reflexive monitoring looks at the ability to look at actions to judge their effectiveness in achieving their objectives: if agents can reproduce structure through action, they can also transform it.

 

Q. According to the author, possession of agency allows which of the following?

Detailed Solution for CAT Practice Test - 26 - Question 21

Option 1 convulates the meaning of the data provided in the passage. The passage states that “agents’ knowledge of the society informs his action”.
Option 2 elaborates on ‘ontological security’ and not ‘possession of agency’.
Option 3 is incorrect because the possession of agency might lead to an instigation of shift in the social structure, not ‘breaking of shifts’.
The last paragraph of the passage states “ ....... as the possession of agency allows one to break away from normative actions, and depending ......... instigate shifts in the social structure”.
Hence, the correct answer is option 4.

CAT Practice Test - 26 - Question 22

Structuration theory aims to avoid extremes of structural or agent determinism. The balancing of agency and structure is referred to as the duality of structure: social structures make social action possible and at the same time, that social action creates those very structures. For Giddens, structures are rules and resources (sets of transformation relations) organized as properties of social systems. Rules are patterns people may follow in social life. Resources relate to what is created by human action; they are not given by nature (explained further below). The theory employs a recursive notion of actions constrained and enabled by structures which are produced and reproduced by those actions. Consequently, this theory has been adopted by those with structuralist inclinations, but who wish to situate such structures in human practice rather than reify them as an ideal type or material property. (This is different, for example, from actor-network theory which grants certain autonomy to technical artefacts.) Additionally, the theory of structuration distinguishes between discursive and practical knowledge recognizes actors as having knowledge is reflexive and situated, and that habitual use becomes institutionalized.
A social system can be understood by its structure, modality, and interaction. Structure is constituted by rules and resources governing and available to agents. (Authoritative resources control persons, whereas allocative resources control material objects.) The modality of a structural system is the means by which structures are translated into action. Interaction is the activity instantiated by the agent acting within the social system. There has been some attempt by various theorists to link structuration theory to systems theory (with its emphasis on recursive loops) or the complexity theory of organizational structure (which emphasizes the adaptability that simple structures provide). Social systems have patterns of social relation that exist over time; the changing nature of space and time will determine the interaction of social relations and therefore structure. For example, 19th century Britain set out certain rules for that time and space. Those rules affected the action which determines structure and the structure was upheld as long as it was reproduced in action. Hitherto social structures or ‘models of society’ were taken to be beyond the realm of human control - the positivistic approach; the other social theory would be that of action creating society - the interpretivist approach. The duality of structure would argue that, in the most basic assumption, that they are one and the same - different sides to the coin of a similar problem of order.
Agency, as Giddens calls it, is human action. To be human is to be an agent, although not all agents are human beings. Agents’ knowledge of their society informs their action, which reproduce social structures, which in turn enforce and maintain the dynamics of action. Giddens defines ‘ontological security’ as the trust people have in social structure; everyday actions have some degree of predictability, thus ensuring social stability. This is not always true, though, as the possession of agency allows one to break away from normative actions, and depending on the sum of social factors at work, they may instigate shifts in the social structure. The dynamic between agency and structure makes such generative action possible. Thus agency can lead to both the reproduction and the transformation of society. Another way to explain this concept is by, what Giddens calls, the “reflexive monitoring of actions”. Reflexive monitoring looks at the ability to look at actions to judge their effectiveness in achieving their objectives: if agents can reproduce structure through action, they can also transform it.

 

Q. The passage states all of the following except:

Detailed Solution for CAT Practice Test - 26 - Question 22

Options 1 and 2 are mentioned in the passage. The passage states, “A social system can be understood by its structure, modality, and interaction” and “the dynamic between agency and structure makes such generative action possible”.
Option 4 is also mentioned in the “reflexive monitoring looks at the ability to look at actions to judge their effectiveness in achieving their objectives: if agents can reproduce structure through action, they can also transform it” and “social structures....similar problem of order”.
Though the author mentions that all humans are agents, he also specifies that all agents are not necessarily human beings. Hence, the correct answer is option 3.

CAT Practice Test - 26 - Question 23

Structuration theory aims to avoid extremes of structural or agent determinism. The balancing of agency and structure is referred to as the duality of structure: social structures make social action possible and at the same time, that social action creates those very structures. For Giddens, structures are rules and resources (sets of transformation relations) organized as properties of social systems. Rules are patterns people may follow in social life. Resources relate to what is created by human action; they are not given by nature (explained further below). The theory employs a recursive notion of actions constrained and enabled by structures which are produced and reproduced by those actions. Consequently, this theory has been adopted by those with structuralist inclinations, but who wish to situate such structures in human practice rather than reify them as an ideal type or material property. (This is different, for example, from actor-network theory which grants certain autonomy to technical artefacts.) Additionally, the theory of structuration distinguishes between discursive and practical knowledge recognizes actors as having knowledge is reflexive and situated, and that habitual use becomes institutionalized.
A social system can be understood by its structure, modality, and interaction. Structure is constituted by rules and resources governing and available to agents. (Authoritative resources control persons, whereas allocative resources control material objects.) The modality of a structural system is the means by which structures are translated into action. Interaction is the activity instantiated by the agent acting within the social system. There has been some attempt by various theorists to link structuration theory to systems theory (with its emphasis on recursive loops) or the complexity theory of organizational structure (which emphasizes the adaptability that simple structures provide). Social systems have patterns of social relation that exist over time; the changing nature of space and time will determine the interaction of social relations and therefore structure. For example, 19th century Britain set out certain rules for that time and space. Those rules affected the action which determines structure and the structure was upheld as long as it was reproduced in action. Hitherto social structures or ‘models of society’ were taken to be beyond the realm of human control - the positivistic approach; the other social theory would be that of action creating society - the interpretivist approach. The duality of structure would argue that, in the most basic assumption, that they are one and the same - different sides to the coin of a similar problem of order.
Agency, as Giddens calls it, is human action. To be human is to be an agent, although not all agents are human beings. Agents’ knowledge of their society informs their action, which reproduce social structures, which in turn enforce and maintain the dynamics of action. Giddens defines ‘ontological security’ as the trust people have in social structure; everyday actions have some degree of predictability, thus ensuring social stability. This is not always true, though, as the possession of agency allows one to break away from normative actions, and depending on the sum of social factors at work, they may instigate shifts in the social structure. The dynamic between agency and structure makes such generative action possible. Thus agency can lead to both the reproduction and the transformation of society. Another way to explain this concept is by, what Giddens calls, the “reflexive monitoring of actions”. Reflexive monitoring looks at the ability to look at actions to judge their effectiveness in achieving their objectives: if agents can reproduce structure through action, they can also transform it.

 

Q. What is the similarity between the actions of the agency and reflexive monitoring?

Detailed Solution for CAT Practice Test - 26 - Question 23

In the passage, it is observed that actions of the agency and reflexive monitoring both are used essentially for bringing about a transformation of the structure.
The passage states “Thus agency can lead to both the reproduction and the transformation of society. Another way to explain this concept.... the “ reflexive monitoring of actions” .
Reflexive monitoring ...... ability to look at actions to judge ................through action, they can also transform it.” The idea has been captured in option 4.
Option 1 is not related to the action of the agency and is thus incorrect.
Options 2 and 3 have not been mentioned in the passage. Hence, the correct answer is option 4.

CAT Practice Test - 26 - Question 24

Structuration theory aims to avoid extremes of structural or agent determinism. The balancing of agency and structure is referred to as the duality of structure: social structures make social action possible and at the same time, that social action creates those very structures. For Giddens, structures are rules and resources (sets of transformation relations) organized as properties of social systems. Rules are patterns people may follow in social life. Resources relate to what is created by human action; they are not given by nature (explained further below). The theory employs a recursive notion of actions constrained and enabled by structures which are produced and reproduced by those actions. Consequently, this theory has been adopted by those with structuralist inclinations, but who wish to situate such structures in human practice rather than reify them as an ideal type or material property. (This is different, for example, from actor-network theory which grants certain autonomy to technical artefacts.) Additionally, the theory of structuration distinguishes between discursive and practical knowledge recognizes actors as having knowledge is reflexive and situated, and that habitual use becomes institutionalized.
A social system can be understood by its structure, modality, and interaction. Structure is constituted by rules and resources governing and available to agents. (Authoritative resources control persons, whereas allocative resources control material objects.) The modality of a structural system is the means by which structures are translated into action. Interaction is the activity instantiated by the agent acting within the social system. There has been some attempt by various theorists to link structuration theory to systems theory (with its emphasis on recursive loops) or the complexity theory of organizational structure (which emphasizes the adaptability that simple structures provide). Social systems have patterns of social relation that exist over time; the changing nature of space and time will determine the interaction of social relations and therefore structure. For example, 19th century Britain set out certain rules for that time and space. Those rules affected the action which determines structure and the structure was upheld as long as it was reproduced in action. Hitherto social structures or ‘models of society’ were taken to be beyond the realm of human control - the positivistic approach; the other social theory would be that of action creating society - the interpretivist approach. The duality of structure would argue that, in the most basic assumption, that they are one and the same - different sides to the coin of a similar problem of order.
Agency, as Giddens calls it, is human action. To be human is to be an agent, although not all agents are human beings. Agents’ knowledge of their society informs their action, which reproduce social structures, which in turn enforce and maintain the dynamics of action. Giddens defines ‘ontological security’ as the trust people have in social structure; everyday actions have some degree of predictability, thus ensuring social stability. This is not always true, though, as the possession of agency allows one to break away from normative actions, and depending on the sum of social factors at work, they may instigate shifts in the social structure. The dynamic between agency and structure makes such generative action possible. Thus agency can lead to both the reproduction and the transformation of society. Another way to explain this concept is by, what Giddens calls, the “reflexive monitoring of actions”. Reflexive monitoring looks at the ability to look at actions to judge their effectiveness in achieving their objectives: if agents can reproduce structure through action, they can also transform it.

 

Q. Which of the following supports the theory that duality of structure is cyclic in nature?

Detailed Solution for CAT Practice Test - 26 - Question 24

The passage states that the structure leads to social action and this in turn leads to a structure. Thus, this relationship between the action and structure describes the cyclic nature of structure.
The exclusivity of agency and structure is not connected to the cyclic property of duality of structure and thus option 1 is incorrect.
Options 2 and 3 though mentioned in the passage do not account for the cyclic nature of the structure and action and thus are incorrect.
Hence, the correct answer is option 4.

*Answer can only contain numeric values
CAT Practice Test - 26 - Question 25

Fill in the blanks with the most appropriate pair of words from the given options.

Advances in the numeral system and mathematical notation _______ led to the discovery of m athematical_________such as addition, multiplication, squaring, square root, and so forth.

1. finally, functions
2. fundamentally, calculations
3. eventually, operations
4. eventually, concepts


Detailed Solution for CAT Practice Test - 26 - Question 25

Advances (as an on-going process) in the numeral system... “eventually led” is logically more coherent than “fundamentally”and ‘finally led”. This eliminates point 1 and 2.
Addition, subtraction etc. are mathematical “functions” or “operations”. They are not “concepts”. We can eliminate point 4.
Hence, the correct answer is 3.

*Answer can only contain numeric values
CAT Practice Test - 26 - Question 26

Arrange the given sentences in the most logical sequence.

1. In particular, they are paying too little attention to the true causes of (and potential solutions to) the loss of species around the world - a massive die-off often referred to as ‘the sixth extinction’.
2. In focusing on the staggering planetary impacts of greenhouse emissions, they are losing sight of the other ways that human beings lay a heavy hand on the planet.
3. The immense challenge of climate change has caused myopia among a lot of politicians, sending them into a self-destructive state of denial.
4. More quietly, though, that immensity has triggered another kind of myopia, this one among conservationists.


Detailed Solution for CAT Practice Test - 26 - Question 26

The paragraph is an extract about climate change and its impact on the animals. Only sentence 3 fits as the first statement, followed by sentence 4 which talk about “another kind of myopia”. In sentence 2, “they” refers to the conservationists in the previous statement. Finally, statement 1 aptly concludes the passage.
Hence, the correct sequence is 3421.

*Answer can only contain numeric values
CAT Practice Test - 26 - Question 27

Arrange the given sentences in the most logical sequence.

1. You can add your computer to the network, if you so wish.
2. Imagine an enormous digital record. Anyone with internet access can look at the information within: it is open for all to see.
3. The computers’ owners choose to add their machines to the network because, in exchange for their computer’s services, they sometimes receive payment.
4. Nobody is in charge of this record. It is not maintained by a person, a company or a government department, but by 8,000-9,000 computers at different locations around the world in a distributed network.


Detailed Solution for CAT Practice Test - 26 - Question 27

The passage talks about digital record keeping. Only sentence 2 fits as an apt opening statement to the passage. This is followed by statement 4, “this record” refers to the ‘digital record’ mentioned in the previous statement. Statement 3 and 1 form a pair. Statement 3 states why an individual would add their computer to the network. Finally, statement 1 tries to woe the reader into joining the network and appropriately ends the passage.
Hence, the correct sequence is 2431.

*Answer can only contain numeric values
CAT Practice Test - 26 - Question 28

Choose the odd one out from the sentences given below.

1. Demand is rising fast as the Gulf countries are reducing their dependence on imported soil and compost made from wastes.
2. The buyers in Saudi Arabia mistook it for hashish and rejected it despite the fact that the buying agency based in the country had several good contacts.
3. Coir pith, or the spongy material inside the coconut husk, is exported mainly from Kerala and Tamil Nadu, the top coconut producing states.
4. Gulf countries have long funded Kerala's riches, but of late, God's Own Country has been increasingly returning the favour, supplying an alternative medium for growing flowers, vegetables and the ubiquitous date palms in a region where soil is scarce.


Detailed Solution for CAT Practice Test - 26 - Question 28

The statements 4-3-1 in that order tell us about the export of coir pith by Kerala and Tamil Nadu to Gulf countries where it is used as “...an alternative medium for ...(soil)... where soil is scarce.”. The specific incident in Saudi Arabia described in statement 2 is out of context. It goes against the point being made in 4 about Gulf countries funding Kerala's riches.
Hence, the correct answer is 2.

*Answer can only contain numeric values
CAT Practice Test - 26 - Question 29

Choose the odd one out from the sentences given below.

1. But it is also puzzling.
2. As if this wasn’t bad enough, the banks reportedly will be able to use government money to write down the value of mortgages, which amounts to subsidizing them to pay their own meaningless fines.
3. Why would the administration continue to bend over backwards to be lenient towards top bankers under these circumstances?
4. In fact, the Obama administration’s settlement with the mortgage lenders is consistent with its track record on all of its policies related to the financial sector, which has been abysmal.


Detailed Solution for CAT Practice Test - 26 - Question 29

Statement 2 is the odd one out. Statements 4-1-3 in that order inform us about the author’s puzzlement about the administration’s abysmal policies related to the financial sector. Statement 2 which begins with “As if all this wasn’t bad enough...” and only pertains to banking does not refer to anything in the other three statements. Hence, the correct answer is 2.

*Answer can only contain numeric values
CAT Practice Test - 26 - Question 30

The following question consists of a set of labelled sentences. These sentences, when properly sequenced, form a coherent paragraph. Choose the most logical order of sentences from the options.

1. If it's a real dream, you cannot control it.
2. I can continue the next day, if it’s a real dream, you can’t do that.
3. When writing a book, you are awake; you can choose the time, the length, everything.
4. The good thing about writing books is that you can dream while you are awake.
5. I write for four or five hours in the morning and when the time comes, I stop.


Detailed Solution for CAT Practice Test - 26 - Question 30

The statements discuss the author's experience of writing books. In statement 4, the author highlights the good thing about writing books which is the ability to dream while awake. This is followed by statement 1 which gives the drawback of real dream. The "time" mentioned in statement 3 is elaborated on with the mention of "four or five hours" in statement 5. The phrase "I can continue" in statement 2 relates to "I write for four or five hours" in statement 5. Thus, 2 follows 5.
Hence, the correct sequence is 41352.

View more questions
1 videos|75 docs|469 tests
Information about CAT Practice Test - 26 Page
In this test you can find the Exam questions for CAT Practice Test - 26 solved & explained in the simplest way possible. Besides giving Questions and answers for CAT Practice Test - 26, EduRev gives you an ample number of Online tests for practice

Top Courses for CAT

Download as PDF

Top Courses for CAT